Đến nội dung

Nguyenhuyen_AG nội dung

Có 785 mục bởi Nguyenhuyen_AG (Tìm giới hạn từ 27-04-2020)



Sắp theo                Sắp xếp  

#690381 cho x;y>0 thỏa mãn x3+y3=x-y

Đã gửi bởi Nguyenhuyen_AG on 12-08-2017 - 22:51 trong Bất đẳng thức - Cực trị

Cho x;y >0 thỏa mãn x3+y3=x-y tìm max P=$x^{2}+(2+2\sqrt{2})y^{2}$

 

Giá trị lớn nhất là $1.$




#690379 $1+\frac{1}{x^2}$ ≥ $cosA+x(cosB+cosC...

Đã gửi bởi Nguyenhuyen_AG on 12-08-2017 - 22:48 trong Hình học phẳng

 

Cho tam giác ABC là một tam giác bất kì. Chứng minh rằng với mọi số x ta đều có

 

$1+\frac{1}{2}x^2$ ≥ $cosA+x(cosB+cosC)$

 

 

 

Mình có một cách dùng tam thức bậc hai nhưng khá xấu.




#690337 Tìm giá trị lớn nhất và nhỏ nhất của biểu thức E = 2xy - y^2

Đã gửi bởi Nguyenhuyen_AG on 12-08-2017 - 18:39 trong Bất đẳng thức và cực trị

Còn GTLN ạ...

 

Ta có

\[2xy-y^2 = \frac14(x^2+2xy+5y^2-8)-\frac14(x-3y)^2+2 \leqslant 2.\]

Đẳng thức xảy ra khi $x=3y.$




#690336 $\sum \frac{a}{a+b}+\frac{abc...

Đã gửi bởi Nguyenhuyen_AG on 12-08-2017 - 18:27 trong Bất đẳng thức và cực trị

Lời giải của hanguyen445 sai vì bất đẳng thức $(*)$ không đúng. Bài này có thể chứng minh bằng Cauchy-Schwarz lúc trước có một bạn đã đăng lên diễn đàn và mình cũng có đăng một bài mạnh hơn của nó.




#690289 $a + b + 2ab \le \frac{3}{2}$

Đã gửi bởi Nguyenhuyen_AG on 11-08-2017 - 23:25 trong Bất đẳng thức và cực trị

Cho  a,b > 0 thỏa ${a^2} + {b^2} + 8{a^2}{b^2} = 1$

Chứng minh rằng :  $a + b + 2ab \le \frac{3}{2}$

 

Ta có

\[\frac{3}{2} - (a + b + 2ab) = \frac{(32ab^2+4a-6b-3)^2+\left[2(4b-1)^2+9\right](2b-1)^2}{24(8b^2+1)} \geqslant 0.\]

 

Thêm một bài mới tương tự:

Cho $a,b > 0$ thỏa $\frac{5}{2}({a^2} + {b^2}) + 500{a^2}{b^2} = 1$  . CMR:

$a + b + 2017ab \le \frac{{2027}}{{25}}$

 

Đặt $P = \frac{{2027}}{{25}} -(a + b + 2017ab),$ thì

\[P = \frac{25(134800ab^2+674a-6051b-3)^2+(90855200b^2+35533280b+3654203)(5b-1)^2}{101100(200b^2+1)} \geqslant 0.\]




#690263 Bất đẳng thức lượng giác

Đã gửi bởi Nguyenhuyen_AG on 11-08-2017 - 21:32 trong Bất đẳng thức - Cực trị

À, đúng rồi, thank anh nhé. Mà cái này cũng hơi bị "cơ bản" đấy ạ  :icon6:  :icon6:  :icon6:  :icon6:

 

Anh nhớ nó cơ bản mà ta. :closedeyes:




#690248 Cho $a,b,c>0$ thỏa $ab+bc+ca=\frac{1}{...

Đã gửi bởi Nguyenhuyen_AG on 11-08-2017 - 20:34 trong Bất đẳng thức và cực trị

Cho $a,b,c>0$ thỏa $ab+bc+ca=\frac{1}{3}$. CM: $\sum \frac{1}{a^2-bc+1} \le 3$

 

Bài này là China TST 2005.




#690247 Bất đẳng thức lượng giác

Đã gửi bởi Nguyenhuyen_AG on 11-08-2017 - 20:29 trong Bất đẳng thức - Cực trị

Anh cho em tham khảo chứng minh được không ạ?

 

Anh mới online đây, em có thể tham khảo ở đây: https://nguyenhuyena...-thuc-hinh-hoc/ hoặc




#690136 Bất đẳng thức lượng giác

Đã gửi bởi Nguyenhuyen_AG on 10-08-2017 - 18:45 trong Bất đẳng thức - Cực trị

Cái bất đẳng thức $p-2R-r\geq 0$ là sao ạ?

 

Nó là bất đẳng thức cơ bản trong tam giác nhọn.




#690107 Tổng S = a + b + c = 0

Đã gửi bởi Nguyenhuyen_AG on 10-08-2017 - 12:33 trong Bất đẳng thức - Cực trị

Làm vài cách khác đi anh

 

Trong ba số $a,\,b,\,c$ sẽ có hai số cùng dấu, giả sử đó là $a,\,b.$ Khi đó bất đẳng thức cần chứng minh tương đương với

\[\frac{b(b+c)}{c^2} + \frac{a^2}{b^2}+\frac{c^2}{a^2} + \frac{a}{b} + \frac{c}{a} \geqslant \frac{15}{4},\]

hay là

\[-\frac{ab}{(a+b)^2} + \frac{a^2}{b^2}+\frac{(a+b)^2}{a^2} + \frac{a}{b} - \frac{a+b}{a} \geqslant \frac{15}{4},\]

hoặc

\[-\frac{ab}{(a+b)^2} + \frac{a^2}{b^2}+\frac{b^2}{a^2} + \frac{a}{b} + \frac{b}{a} \geqslant \frac{15}{4},\]

\[\left[\frac{1}{4}-\frac{ab}{(a+b)^2}\right]+ \left(\frac{a^2}{b^2}+\frac{b^2}{a^2}-2\right)  + \left(\frac{a}{b} + \frac{b}{a}-2\right)  \geqslant 0.\]

Hiển nhiên đúng theo bất đẳng thức AM-GM.

 




#690004 Chứng minh rằng: $\frac{a+b+c}{3}\geq...

Đã gửi bởi Nguyenhuyen_AG on 09-08-2017 - 17:34 trong Bất đẳng thức - Cực trị

t là gì vậy pạn

 

Ý tưởng này không phải của mình nên xin trích lại nguyên văn

 

$ \frac {a + b + c}{3}\geq \sqrt [27]{\frac {a^3 + b^3 + c^3}{3}}\Longleftrightarrow (a + b + c)^{27}\geq 3^{26}(a^3 + b^3 + c^3)\ \cdots [*]$.
Let $ a + b = 2x,\ b + c = 2y,\ c + a = 2z$, we have that $ (a + b)(b + c)(c + a) = 8\Longleftrightarrow xyz = 1$ and $ 2(a + b + c) = 2(x + y + z)\Longleftrightarrow a + b + c = x + y + z$.$ (a + b + c)^3 = a^3 + b^3 + c^3 + 3(a + b)(b + c)(c + a)\Longleftrightarrow a^3 + b^3 + c^3 = (x + y + z)^3 - 24$. $ \therefore [*]\Longleftrightarrow (x + y + z)^{27}\geq 3^{26}\{(x + y + z)^3 - 24\}$.
Let $ t = (x + y + z)^3$, by AM-GM, we have that $ x + y + z\geq 3\sqrt [3]{xyz}\Longleftrightarrow x + y + z\geq 3$, yielding $ t\geq 27$.
Since $ y = t^9$ is an increasing and concave up function for $ t > 0$, the tangent line of $ y = t^9$ at $ t = 3$ is $ y = 3^{26}(t - 27) + 3^{27}$.We can obtain $ t^9\geq 3^{26}(t - 27) + 3^{27}$, yielding $ t^9\geq 3^{26}(t - 24)$, which completes the proof.

 

Với cách đặt này thì khi $t^9 \geqslant 3^{26}(t - 27) + 3^{27} = 0$ sẽ có $t=27.$

 

Maple đó bạn, nhưng phải có code nữa

 

Để phân tích $t^9 - 3^{26}(t - 27) - 3^{27}$ thành nhân tử chỉ một lệnh factor.




#689983 Chứng minh rằng: $\frac{a+b+c}{3}\geq...

Đã gửi bởi Nguyenhuyen_AG on 09-08-2017 - 14:12 trong Bất đẳng thức - Cực trị

Bạn nào có cách giải khác dùng biến đổi tương đương k ạ

 

Có một cách biến đối tương đương nhưng bất đẳng thức cuối cùng thu được rất khủng khiếp

\[(t^7+54t^6+2187t^5+78732t^4+2657205t^3+86093442t^2+2711943423t+83682825624)(t-27)^2 \geqslant 0.\]




#689982 Tổng S = a + b + c = 0

Đã gửi bởi Nguyenhuyen_AG on 09-08-2017 - 14:08 trong Bất đẳng thức - Cực trị

Không có cách " đẹp " hơn à ?

 

Có 7 cách.




#689954 cho 3 số x, y, z thỏa mãn: x +y +z =0 và x^2 +y^2 +z^2 =a^2. TÍnh x^4 +y^4 +z...

Đã gửi bởi Nguyenhuyen_AG on 08-08-2017 - 23:20 trong Đại số

1) Cho 3 số x, y, z thỏa mãn: x +y +z =0 và x^2 +y^2 +z^2 =a^2. Tính x^4 +y^4 +z^4 theo a

 

Đặt $p=x+x+y,q=xy+yz+zx,r=xyz$ từ giả thiết suy ra $q = -\frac{a^2}{2}$ và chú ý

\[x^4+y^4+z^4 = p^4-4p^2q+4pr+2q^2.\]




#689943 bất đẳng thức cauchy

Đã gửi bởi Nguyenhuyen_AG on 08-08-2017 - 22:01 trong Bất đẳng thức và cực trị

Bất đẳng thức này có dạng gì đấy anh ơi

 

Em tìm trên mạng thông tin về bất đẳng thức AM-GM suy rộng.




#689922 Chứng minh rằng: $\frac{a+b+c}{3}\geq...

Đã gửi bởi Nguyenhuyen_AG on 08-08-2017 - 20:20 trong Bất đẳng thức - Cực trị

Cho $a, b, c$ là các số thực dương thỏa $(a+b)(b+c)(c+a)=8.$ Chứng minh rằng: $\frac{a+b+c}{3}\geq \sqrt[27]{\frac{a^{3}+b^{3}+c^{3}}{3}}.$

 

Áp dụng bất đẳng thức cho $9$ số dương ta có

\[(a+b+c)^3 = a^3+b^3+c^3+3(a+b)(b+c)(c+a)=a^3+b^3+c^3+3\cdot 8 \geqslant 9\sqrt[9]{(a^3+b^3+c^3) \cdot 3^8}.\]

Từ đó suy ra điều phải chứng minh.




#689921 bất đẳng thức cauchy

Đã gửi bởi Nguyenhuyen_AG on 08-08-2017 - 20:16 trong Bất đẳng thức và cực trị

Sử dụng bất đẳng thức cô si tìm giá trị nhỏ nhât của S(x)= nx^(n+1)-(n+1)x^n+1

 

Viết bất đẳng thức trên lại như sau

\[\frac{n}{n+1} \cdot x^{n+1}+\frac1{n+1} \cdot 1 \geqslant x^n.\]

Chú ý rằng $\frac{n}{n+1}+\frac1{n+1}=1$ nên bất đẳng thức này đúng theo bất đẳng thức AM-GM suy rộng.




#689920 Bất đẳng thức lượng giác

Đã gửi bởi Nguyenhuyen_AG on 08-08-2017 - 20:12 trong Bất đẳng thức - Cực trị

Cho tam giác ABC nhọn. CMR:

$\tan\frac{A}{2}+tan\frac{B}{2}+tan\frac{C}{2}\leq 2$

 

Ta có

\[\sum \tan\frac{A}{2} = \frac1p \sum r_a = \frac{4R+r}{p},\]

nên bất đẳng thức trên tương đương với

\[2(p-2R-r)+r \geqslant 0.\]

Hiển nhiên đúng do bất đẳng thức trong tam giác nhọn $p-2R-r \geqslant 0.$

 

Anh Huyện có một cách phản chứng khá hay: Giả sử bất đẳng thức trên là sai. Khi đó nó lại mâu thuẫn với yêu cầu bài toán là chứng minh bất đẳng thức trên đúng. Do đó bất đẳng thức trên đúng (dpcm).

 

Em spam ạ!

:lol:




#689919 Tổng S = a + b + c = 0

Đã gửi bởi Nguyenhuyen_AG on 08-08-2017 - 19:49 trong Bất đẳng thức - Cực trị

Cho a , b , c đôi một khác nhau thỏa a + b + c = 0 . Cmr

$\sum \frac{a^{2}}{b^{2}} + \sum \frac{a}{b}\geqslant \frac{15}{4}$

 

Sao lại cần điều kiện đôi một khác nhau nhỉ?

 

Thay $c = -a-b$ bất đẳng thức trên trở thành

\[\frac{a^2}{b^2} + \frac{b^2}{(a+b)^2} + \frac{(a+b)^2}{a^2} + \frac{a}{b} - \frac{b}{a+b} - \frac{a+b}{a} \geqslant \frac{15}{4}.\]

Đặt $a=kb$ bất đẳng thức trên tương đương với

\[k^2 + \frac{1}{(k+1)^2} + \frac{(k+1)^2}{k^2} + k - \frac{1}{k+1} - \frac{k+1}{k} \geqslant \frac{15}{4},\]

hoặc

\[\frac{(k-1)^2(2k+1)^2(k+2)^2}{4k^2(k+1)^2} \geqslant 0.\]

Hiển nhiên đúng.




#689124 $$ \sum_{cyc} \sqrt{ \dfrac{a...

Đã gửi bởi Nguyenhuyen_AG on 30-07-2017 - 21:54 trong Bất đẳng thức - Cực trị

anh dùng phần mềm gì phân tích cái cuối thế ạ, cho em xin link tải với :3

 

Anh dùng Maple để tách hệ số, nhưng phải có code riêng do anh viết mới phân tích được như thế chứ maple thuần túy không làm được.




#689096 $$ \sum_{cyc} \sqrt{ \dfrac{a...

Đã gửi bởi Nguyenhuyen_AG on 30-07-2017 - 18:35 trong Bất đẳng thức - Cực trị

Cho $a,b,c >0$ thỏa $a+b+c=3$. Chứng minh rằng
$$ \sum_{cyc} \sqrt{ \dfrac{a}{3b^{2}+1}} \geq \dfrac{3}{2}$$
Trịnh Đình Triển (Dinh De Tai) nhờ mình post bài này.

 

Áp dụng bất đẳng thức Holder, ta có

\[\left(\sum \sqrt{ \dfrac{a}{3b^{2}+1}}\right)^2 \sum a^2(3b^2+1)(a+b+3c)^3 \geqslant \left[\sum a(a+b+3c)\right]^3.\]

Như vậy ta cần chứng minh

\[\left[\sum a(a+b+3c)\right]^3 \geqslant \frac94\sum a^2(3b^2+1)(a+b+3c)^3,\]

hay là

\[\frac13\left[\sum a(a+b+3c)\right]^3(a+b+c) \geqslant \frac94\sum a^2\left[3b^2+\frac19(a+b+c)^2\right] (a+b+3c)^3,\]

\[\begin{aligned}\sum \frac{1}{132}a\left[11a^6+11(37b+19c)a^5+2(127b^2+2178bc+165c^2)a^4+308c^3a^3+440c^4a^2+7c^4(735b+58c)a\\+b^2c^2(2709b^2+7898bc+7788c^2)-30261ab^2c^3\right]  \geqslant  0.\end{aligned}\]

Bất đẳng thức cuối cùng đúng theo bất đẳng thức AM-GM suy rộng.




#688624 Cmr:$\sum\frac{x+y-z}{\sqrt{z^2+(x-y)...

Đã gửi bởi Nguyenhuyen_AG on 25-07-2017 - 14:01 trong Bất đẳng thức và cực trị

Chứng minh sao ạ

 

Anh tính nhầm một chỗ nên bất đẳng thức anh nêu ra bị sau, còn bài toán của bạn cristianoronaldo ta có thể chứng minh như sau

 

Dùng phép thế Ravi ta đưa bài toán về dạng

\[\sum \frac{c}{\sqrt{a^2+b^2}} \geqslant \frac{\sqrt2(a+b+c)^2}{a^2+b^2+c^2+ab+bc+ca}, \quad \forall a,b,c > 0.\]

Bất đẳng thức này được suy ra từ hai đánh giá liên tiếp sau đây

\[\left(\sum \frac{c}{\sqrt{a^2+b^2}}\right)^2 \geqslant \frac{(a+b+c)^3}{\displaystyle \sum c(a^2+b^2)} \geqslant \frac{2(a+b+c)^4}{(a^2+b^2+c^2+ab+bc+ca)^2}.\]

Vế trái chính là bất đẳng thức Holder còn vế phải sau khi khai triển và thu gọn sẽ được một bất đẳng thức hiển nhiên $\sum a^4 \geqslant \sum b^2c^2.$




#688558 Cmr:$\sum\frac{x+y-z}{\sqrt{z^2+(x-y)...

Đã gửi bởi Nguyenhuyen_AG on 24-07-2017 - 21:33 trong Bất đẳng thức và cực trị

$\boxed{\text{Bài toán}}$:

Cho x,y,z là độ dài ba cạnh tam giác. Chứng minh rằng:

$\frac{x+y-z}{\sqrt{z^2+(x-y)^2}}+\frac{y+z-x}{\sqrt{x^2+(y-z)^2}}+\frac{z+x-y}{\sqrt{y^2+(z-x)^2}}\geq \frac{(x+y+z)^2}{x^2+y^2+z^2}$

 

Bất đẳng thức chặt hơn vẫn đúng

\[\frac{x+y-z}{\sqrt{z^2+(x-y)^2}}+\frac{y+z-x}{\sqrt{x^2+(y-z)^2}}+\frac{z+x-y}{\sqrt{y^2+(z-x)^2}} \geqslant 3.\]




#688344 $\sum \frac{ab}{a^2+3b^2}\geq \frac{3}{4}$

Đã gửi bởi Nguyenhuyen_AG on 22-07-2017 - 15:29 trong Bất đẳng thức và cực trị

Ơ, em xin lỗi, hay là của thầy Quốc Anh ạ?

 

Bài này là của anh Dương Đức Lâm.




#688343 $\sum \frac{a^2+bc}{b^2+bc+c^2}\geq 2$

Đã gửi bởi Nguyenhuyen_AG on 22-07-2017 - 15:23 trong Bất đẳng thức và cực trị

The problem $2$ can make a proof easily by using $p,q,r$ changing variables technique combines $Schur's$ $inequality$. You see that.

 

Bài này dồn biến sẽ là lựa chọn tốt nhất. :D